Expressing a vector in the exponential form

Click For Summary
The discussion focuses on expressing vectors in exponential form using complex numbers. The participants explore equations involving trigonometric identities and their relationships to complex exponentials, specifically using the real part of expressions. Techniques for simplifying complex expressions are highlighted, including the use of cosine and sine in relation to exponential functions. The conversation emphasizes the importance of reducing complex terms to achieve simpler forms. Overall, the thread illustrates methods for manipulating and understanding complex expressions in the context of vector representation.
LCSphysicist
Messages
644
Reaction score
162
Homework Statement
All below
Relevant Equations
All below
1594070336690.png

1594070362759.png

1594070351172.png

I managed to expand a general expression from the alternatives that would leave me to the answer, that is:
I will receive the alternatives like above, so i find the equation:

1594070509099.png
C = -sina, P = cosa

So reducing B:
1594070587524.png


R:
1594070730178.png
Reducing D:
1594070763098.png


R:
1594070886950.png


Is this right?
 
Physics news on Phys.org
I prefer simply the way
(b) ##\ \ Re(e^{i(\omega t-\pi/3)}-e^{i\omega t})=Re(e^{i\omega t}(e^{-i\pi/3}-1))=Re(e^{i\omega t} e^{-i2\pi/3})##

(d) ##\ \ Re(e^{i\omega t-i \pi/2}-2e^{i(\omega t - \pi/4 )}+e^{i\omega t})##
 
Last edited:
  • Like
Likes LCSphysicist and etotheipi
The idea is to use the fact that ##\cos \alpha = {\rm Re}(e^{i\alpha})##, then simplify the complex expression.

One technique you can use is
\begin{align*}
e^{i\theta} + 1 &= e^{i\theta/2}(e^{i\theta/2} + e^{-i\theta/2}) = e^{i\theta/2}[2 \cos (\theta/2)] \\
e^{i\theta} - 1 &= e^{i\theta/2}(e^{i\theta/2} - e^{-i\theta/2}) = e^{i\theta/2}[2i \sin (\theta/2)]
\end{align*} or some variation.
 
  • Like
  • Skeptical
Likes LCSphysicist and etotheipi
Question: A clock's minute hand has length 4 and its hour hand has length 3. What is the distance between the tips at the moment when it is increasing most rapidly?(Putnam Exam Question) Answer: Making assumption that both the hands moves at constant angular velocities, the answer is ## \sqrt{7} .## But don't you think this assumption is somewhat doubtful and wrong?

Similar threads

  • · Replies 23 ·
Replies
23
Views
3K
Replies
6
Views
1K
  • · Replies 2 ·
Replies
2
Views
3K
Replies
11
Views
2K
Replies
4
Views
2K
Replies
6
Views
1K
Replies
4
Views
2K
Replies
4
Views
2K
  • · Replies 7 ·
Replies
7
Views
2K
Replies
3
Views
2K